Đến nội dung

Stranger411 nội dung

Có 85 mục bởi Stranger411 (Tìm giới hạn từ 29-04-2020)



Sắp theo                Sắp xếp  

#393205 $c \le \frac{1}{4n}$

Đã gửi bởi Stranger411 on 04-02-2013 - 21:12 trong Dãy số - Giới hạn

hình như chỗ

hình như chỗ này có vấn đề

Đúng rồi bạn à :)
có $a_n =0$ mới có thể suy ra được công thức truy hồi giữa $a_k$ và $a_{k+1}$

ps: nói thế là gần hết bài toán rồi @@!



#392825 Đếm bằng $v_p$

Đã gửi bởi Stranger411 on 03-02-2013 - 15:47 trong Tổ hợp và rời rạc

Cho bảng hình chữ nhật kích thước $m$ x $n$ ($m,n \in \mathbb{Z^+}$) và các số nguyên dương $p \le m$ và $q \le n$. Người ta điền các số $1,2,3, ..., m$ x $n$ vào các ô trong bảng. Một số tự nhiên trong bảng được gọi là "xấu" nếu nó nhỏ hơn $p$ số cùng cột và $q$ số cùng hàng. Đếm số cách xếp sao cho các số xấu có trong bảng là nhỏ nhất và hãy chỉ ra 1 cách xếp như vậy.



Ps: Đang viết 1 bài về $v_p$ và đây là một ứng dụng trong tổ hợp của nó.



#392819 Bất khả quy trên $\mathbb{Z[x]}$

Đã gửi bởi Stranger411 on 03-02-2013 - 15:29 trong Đa thức

Cho đa thức $P(x)= \prod_{i=1}^{n}( x- a_i)$ với $n \ge 5$ và các $a_i \in \mathbb{Z}$ phân biệt. Chứng minh nếu tam thức $ a x^2 + bx +1$ bất khả quy trên $\mathbb{Z[x]}$ thì đa thức $ a P(x)^2 + b P(x) +1$ cũng bất khả quy trên $\mathbb{Z[x]}$



#392817 $c \le \frac{1}{4n}$

Đã gửi bởi Stranger411 on 03-02-2013 - 15:17 trong Dãy số - Giới hạn

Cho dãy số $a_1, a_2, ..., a_n$ và số $c$ thỏa mãn 2 điều kiện:
1) $a_n =0$
2) $a_k = c+ \sum_{i=k}^{n-1}(a_i + a_{i+1})$ $\forall k=\overline{0,n-1}$.
Chứng minh $c \le \frac{1}{4n}$



#392813 $a^2 +b | a + b^2$

Đã gửi bởi Stranger411 on 03-02-2013 - 14:57 trong Số học

Cho số nguyên tố $p$, tìm tất cả các cặp số nguyên dương $(a,b)$ thỏa 2 điều kiện:
1) $a^2 +b = p^k$ với $k \in \mathbb{Z^+}$
2) $a^2 +b | a + b^2$


ps: mấy chú ở Khtn chắc biết bài này nhỉ ;)



#387048 $P(x^2 -2002) \vdots P(x)$

Đã gửi bởi Stranger411 on 15-01-2013 - 22:00 trong Đa thức

Có tồn tại hay không đa thức P(x) thỏa:
(1) $degP(x) = 2003$
(2) $P(x^2 -2002) \vdots P(x)$ ($\forall x \in \mathbb{R}$)


ps: hồi chiều làm kiểm tra bị bể bài này :((((



#385774 VMO 2013 - Bài 1. Hệ phương trình

Đã gửi bởi Stranger411 on 11-01-2013 - 23:00 trong Phương trình - Hệ phương trình - Bất phương trình

Cách khác: (không dùng Minkowski :D)
ĐKXĐ: $x\neq \dfrac{m\pi }{2}$, $y\neq \dfrac{n\pi }{2}$ ($m$, $n\in \mathbb{Z}$), $xy\geq 0$, $x+y\neq 0$.
Ta chứng minh:
$(\sqrt{{{\sin }^{2}}x+\frac{1}{{{\sin }^{2}}x}}+\sqrt{{{\cos }^{2}}y+\frac{1}{{{\cos }^{2}}y}})^2 + (\sqrt{{{\sin }^{2}}y+\frac{1}{{{\sin }^{2}}y}}+\sqrt{{{\cos }^{2}}x+\frac{1}{{{\cos }^{2}}x}})^2 \ge 20$

Áp dụng $\frac{1}{\sin ^2 x} + \frac{1}{\cos ^2 x} \ge 4$ $ \forall x \neq 0$, ta có:
$(\sqrt{{{\sin }^{2}}x+\frac{1}{{{\sin }^{2}}x}}+\sqrt{{{\cos }^{2}}y+\frac{1}{{{\cos }^{2}}y}})^2 + (\sqrt{{{\sin }^{2}}y+\frac{1}{{{\sin }^{2}}y}}+\sqrt{{{\cos }^{2}}x+\frac{1}{{{\cos }^{2}}x}})^2 $
$\ge 10 + 2 \sqrt{{{\sin }^{2}}x+\frac{1}{{{\sin }^{2}}x}}.\sqrt{{{\cos }^{2}}y+\frac{1}{{{\cos }^{2}}y}} + 2 \sqrt{{{\sin }^{2}}y+\frac{1}{{{\sin }^{2}}y}}.\sqrt{{{\cos }^{2}}x+\frac{1}{{{\cos }^{2}}x}}$

Vậy chỉ cần chứng minh:
$ \sqrt{{{\sin }^{2}}x+\frac{1}{{{\sin }^{2}}x}}.\sqrt{{{\cos }^{2}}y+\frac{1}{{{\cos }^{2}}y}} + \sqrt{{{\sin }^{2}}y+\frac{1}{{{\sin }^{2}}y}}.\sqrt{{{\cos }^{2}}x+\frac{1}{{{\cos }^{2}}x}} \ge 5$ $ \forall x \neq 0$
từ đó quay về chứng minh:
$ \left(\sin^2x + \dfrac{1}{\sin^2x}\right)\left(\cos^2x + \dfrac{1}{\cos^2x}\right) \geq \left(\dfrac{5}{2}\right)^2.$và $ \left(\sin^2y + \dfrac{1}{\sin^2y}\right)\left(\cos^2y + \dfrac{1}{\cos^2y}\right) \geq \left(\dfrac{5}{2}\right)^2.$

Sử dụng bất đẳng thức Cauchy-Schwarz và AM-GM ta có
$$ \begin{aligned} \left(\sin^2x + \dfrac{1}{\sin^2x}\right)\left(\cos^2x + \dfrac{1}{\cos^2x}\right) \ & \geq \left(|\sin x\cos x|+\dfrac{1}{|\sin x\cos x|}\right)^2 \\ & =\left(\dfrac{|\sin 2x|}{2}+\dfrac{1}{2|\sin 2x|}+\dfrac{3}{2|\sin 2x|}\right)^2 \\ & \geq \left( 1+\dfrac{3}{2}\right)^2=\left(\dfrac{5}{2}\right)^2.\end{aligned}$$
Ta có (đpcm).
Suy ra: $\tan x = \tan y= \pm 1$ và $x=y$
Từ đó, ta được $ x=y= \dfrac{\pi }{4}+\dfrac{k\pi }{2}$ ($k\in \mathbb{Z}$).



#381518 $p \in A$

Đã gửi bởi Stranger411 on 29-12-2012 - 11:15 trong Số học

Cho tập $A=\left \{ x|x=a^2+2b^2 , a,b\in \mathbb{Z},b \neq 0\right \}$ và 1 số nguyên tố $p$.
Chứng minh rằng nếu $p^2 \in A$ thì $p \in A$



#375898 cho n là số nguyên dương lớn hơn 1. CMR $2^n-1$ không chia hết cho n

Đã gửi bởi Stranger411 on 07-12-2012 - 22:20 trong Số học

cho n là số nguyên dương lớn hơn 1. CMR $2^n-1$ không chia hết cho n

Ý tưởng ko khác gì mấy vs nguyenta98, chủ yếu là xét $v_2$ thôi.
Giải:
Xét phân tích tiêu chuẩn của $n=\prod\limits_{i = 1}^h {{p_i}^{{k_i}}}$ với các số nguyên tố ${p_1} < {p_2} < ... < {p_h}$. Trong đó ${p_i} = 1 + {2^{{r_i}}}{m_i}$ ($m_i$ lẻ) $\Rightarrow n \equiv 1(\bmod {m})$
Đặt $n - 1 = {2^m}t$ $\Rightarrow {2^{{2^m}t}} \equiv - 1(\bmod {p_i})$
Mà $- 1 \equiv {2^{{2^m}t{m_i}}} \equiv {2^{({p_i} - 1)t}} \equiv 1(\bmod {p_i})$
Vậy $p_i = 2$ (Vô lí)
Q.E.D

Remark: Chắc nguyenta98 có biết bài này:
Tìm tất cả số tự nhiên $n$ sao cho: $n|{2^n} + 2$



#375486 $\frac{n}{m}=\sum_{k=1}^{p-...

Đã gửi bởi Stranger411 on 06-12-2012 - 00:19 trong Số học

Cho $p>3$ là một số nguyên tố.

Đặt $\begin{cases}{n\over m}=\sum_{k=1}^{p-1}{1\over k}\\ \mathrm{gcd}(n,m)=1\end{cases}$

Chứng minh rằng: $p\big| n$

Bài này có nhiều cách giải lắm thầy ạ :D
Theo em cách giải ngắn gọn nhất là sử dụng $\mathbb{Z}/p\mathbb{Z}$

Lời giải:
Ta xét bài toán trong $\mathbb{Z}/p\mathbb{Z}$, do tính tồn tại duy nhất của nghich đảo modulo, ta có:
$\overline{1}^{-1} + \overline{2}^{-1} +...+ \overline{p-1}^{-1} = \overline{1} + \overline{2} +...+ \overline{p-1} = \overline{1+2+...+ p-1} = \overline{\frac{p(p-1)}{2}} \equiv 0$
Q.E.D



#375471 $C_{{p^r}}^p \equiv {p^{r - 1}...

Đã gửi bởi Stranger411 on 05-12-2012 - 23:06 trong Số học

CMR: $C_{{p^r}}^p \equiv {p^{r - 1}}(\bmod {p^r})$

Đây là một bài toán không quá khó để phải dùng đến các kiến thức về số ${v_p}$ như vậy.

Lời giải:
Áp dụng công thức $kC_{n}^k =nC_{k-1}^{n-1}$, ta được:
$$C_{p^r}^p - p^{r-1}= p^{r-1} \left ( C_{p^r-1}^{p-1} -1 \right )$$
Từ đó, ta chỉ cần chứng minh:
$$C_{p^r-1}^{p-1} \equiv 1 (mod p) (*)$$
Nhưng đây chỉ là một hệ quả của định lí Lucas.

Remark: Ta có thể chứng minh một bổ đề khác mạnh hơn $(*)$ như sau:

Cho số nguyên tố $p$ và các số tự nhiên $k,a$, trong đó $0 \le a \le p^{k-1}$.
Chứng minh: $C_{p^{k-1}}^{a} \equiv (-1)^a ( mod p)$

Và bổ đề trên cũng chỉ là 1 hệ quả của định lí Lucas



#372277 $Q(k) = p^n$

Đã gửi bởi Stranger411 on 24-11-2012 - 23:22 trong Đa thức

Chứng minh rằng với mọi số nguyên tố $p$, với mọi số nguyên dương $n$, tồn tại đa thức $Q(x) \in \mathbb{z} [x]$ thỏa mãn dãy $Q(1),Q(2),...,Q(n)$ phân biệt và là lũy thừa của $p$.



#372273 Phân hoạch tập tốt

Đã gửi bởi Stranger411 on 24-11-2012 - 23:13 trong Tổ hợp và rời rạc

Cho số nguyên dương $m \ge 3$. Một tập $S$ được gọi là tốt nếu tồn tại những phần tử ${s_1},{s_2},...,{s_m}$ thỏa mãn ${s_1}+{s_2}+...+{s_{m-1}}={s_m}$. Xác định số nguyên dương $f(m)$ nhỏ nhất sao cho một trong 2 phân hoạch A và B của tập ${ 1,2,...,f(m) }$ là 1 tập tốt.



#368914 $\binom{kn}{n}\vdots k^n$

Đã gửi bởi Stranger411 on 12-11-2012 - 08:53 trong Số học

Câu a :
+ $n= 1$ thỏa mãn.
+ $n\geq 2$ qui nạp đơn giản ta đc $\binom{3n}{n}\not\vdots 3^n$
Câu b chắc tương tự.

Bài (a) thực ra có trong THTT nhưng người ta không giải.
Em thử giải rõ ra xem nào :P
Mấy bạn lớp anh cũng nói quy nạp như em mà vào thi có làm ra đâu :P


Gợi ý: Câu (b) khác câu a ở chỗ:
câu (a) quy nạp theo $n$
còn câu (b) quy nạp theo $k$



#368417 $\binom{kn}{n}\vdots k^n$

Đã gửi bởi Stranger411 on 10-11-2012 - 15:31 trong Số học

Tìm tất cả số nguyên $n>1$ sao cho:
a) $\binom{3n}{n}\vdots 3^n$
b) $\binom{kn}{n}\vdots k^n$ với mọi $k>1$.


ps: bài kiểm tra 1 tiết lớp mình :(
có rất ít người làm được cả 2 câu.



#368262 $\sum_{k=0}^{\left\lfloor\frac{n...

Đã gửi bởi Stranger411 on 09-11-2012 - 21:48 trong Tổ hợp và rời rạc

Dạo này box tổ hợp và rời rạc của VMF có vẻ trầm lắng!
Để tránh tình trạng này kéo dài, tôi xin khuấy động bằng một bài nho nhỏ

Cho số nguyên $n\ge 3$. Chứng minh đẳng thức:

$\sum_{k=0}^{\left\lfloor\frac{n}{3}\right\rfloor} \binom{n}{3k} = \dfrac{2^n+(-1)^n\left(3\left\lfloor\frac{n}{3}\right\rfloor-3\left\lfloor\frac{n-1}{3}\right\rfloor-1\right)}{3}$

Chào các anh. Bài này dùng hàm sinh kết hợp với RUF tức là Root of Unity Filter đó ạ :D

Định lí: Cho số nguyên dương $n$. Đặt $k=cos \frac{2 \pi}{n}+i sin \frac{2 \pi}{n}$.
Xét đa thức $f(x)=a_n x^n+a_{n-1} x^{n-1}+...+a_1 x+a_0$. Khi đó:
$a_0 + a_{n} + a_{2n} + ... = \frac{1}{n}[f(1)+f(k)+f(k^2)+...+f(k^{n-1})]$

Bài toán: Đặt $k=cos \frac{2 \pi}{3}+i .sin \frac{2 \pi}{3}$.
Áp dụng định lí trên, ta được:
$\sum_{k=0}^{\left\lfloor\frac{n}{3}\right\rfloor} \binom{n}{3k} = \frac{(1+k)^n + (1+ k^2 )^n + (1+1)^n}{3}$
$=\frac{(-1)^n(k^n + k^{2n}) + 2^n}{3}$
từ đó, ta có kết quả bài toán.

Remark: Ngoài ra, ta còn kết quả khác:
Nếu $n$ chia hết cho 3: $\sum_{k=0}^{\infty } \binom{n}{3k} = \frac{2^n + 2(-1)^n}{3}$
Nếu $n$ không chia hết cho 3: $\sum_{k=0}^{\infty } \binom{n}{3k} = \frac{2^n + (-1)^{n+1}}{3} $


ps hxthanh@: thầy cho em hỏi ạ :D hồi rất lâu rồi ấy,thầy có lập 1 topic tên là "Dãy số đầu năm" trong box Đại số THCS. Sau này mod chuyển đi đâu mất :( Thầy cho em cái link để tham khảo được không ạ, Hồi đó em còn non nên chưa đọc được nhiều :D



#359617 $\frac{{d({n^2})}}{{d(n)...

Đã gửi bởi Stranger411 on 06-10-2012 - 23:04 trong Số học

Với mỗi số nguyên dương n,kí hiệu d(n) là số các ước dương của n.Tìm tất cả số nguyên dương m sao cho tồn tại số nguyên dương thỏa $\frac{{d({n^2})}}{{d(n)}} = m$

Đây là bài 3 của IMO 1998
Các bạn có thể tham khảo một số lời giải khác ở đây:
http://www.artofprob...=124439#p124439



#359606 Tìm nghiệm nguyên

Đã gửi bởi Stranger411 on 06-10-2012 - 22:43 trong Số học

Tìm $p,q\in \mathbb{P}$ thỏa mãn $3pq\mid a^{3pq}-a$ với mọi $a\in \mathbb{Z}^+$

Nếu biết giới hạn $p,q$ và chọn $a$ là căn nguyên thủy của $n$ ngay từ đầu thì bài toán sẽ gọn hơn rất nhiều.
Giả sử $p \ge q$

+ Cho $a=3$, ta có:
${3^{pq}} \equiv 3\left( {\bmod 3pq} \right) \Rightarrow 3\left( {{3^{pq - 1}} - 1} \right) \vdots 3pq \Rightarrow p,q > 3$

+ Cho ${a^{\varphi \left( n \right)}} \equiv 1\left( {\bmod n} \right)$ ( $a$ là căn nguyên thủy của $n$)
Theo định lí Fermat nhỏ: ${a^{p - 1}} \equiv 1\left( {\bmod p} \right)$
Vì ${a^{3pq - 1}} \equiv 1\left( {\bmod p} \right) \Rightarrow p - 1|3pq - 1 \Rightarrow p - 1|3q - 1$
Chứng minh tương tự: $q - 1|3p - 1$
Vì $p \ge q$ nên $3q - 1 \in \left\{ {p - 1;2\left( {p - 1} \right);3\left( {p - 1} \right)} \right\}$
Thay vào điều kiện bài toán, ta được: $\boxed{(p,q)=(11,17),(17,11)}$



#353030 $p \equiv 1 (\bmod m)$

Đã gửi bởi Stranger411 on 08-09-2012 - 23:40 trong Số học

Tổng quát hóa từ một bài toán:
Cho số nguyên tố $p \equiv 1 (\bmod m)$ với $m>2$. Chứng minh:
\[\prod\limits_{i = 1}^p {\left( {{i^{m - 1}} + {i^{m - 2}} + \ldots + i + 1} \right)} \equiv 0 (\bmod p)\]


Bài này dùng 1 tí kiến thức về hệ thu gọn.
Bên mathlink post mấy tháng rồi mà chưa có lời giải :-<



#347148 Tìm các số nguyên dương $a,b,c$ sao cho $\frac{a^{2}+b^{2...

Đã gửi bởi Stranger411 on 16-08-2012 - 10:37 trong Số học

sai từ chỗ này và nguyên nhân là do làm tắt $p|{(2a + b)^2} + 3{b^2}$
$ \Rightarrow \left( {\frac{{ - 3}}{p}} \right) = 1$
muốn dùng lengdre(hay tiếng việ gọi là thặng dư toàn phương) trước tiên ta phải đưa nó về dạng (mà ở đây) là
a2$\equiv$-3 (mod p) cái đã,mà ở đây muốn đưa về dạng này ta phải giả sử a không chia hết cho p,''vậy nên thiếu TH a,b chia hết cho p'',mà TH này luôn đúng,nếu không thấy dc thì cho a=b=p ta có 12p2 chia hết cho p ,vì vậy có giải kiểu gì đi nữa vẫn phải thông qua a,b,c chia hết cho p rồi mới giải tiếp,nên không có cách bạn stranger nói

Nói chuyện vs Uyenha cực kì bực mình @@!
Mình và mọi người đã ko muốn nói rồi mà bạn cứ thích cãi cùn.

Trước đó, MOD đã gộp bớt vài bài của bạn để tránh spam trong topic.
Thắc mắc thì ko phải là tội nhưng cứ nói dai như thế người ta chả thích tí nào đâu bạn :)

Mời bạn tham khảo thêm về kí hiệu Lengdre:
File gửi kèm  Cong Thuc Legendre.pdf   67.83K   1681 Số lần tải



#347133 Tìm số nguyên $n> 1$ sao cho $\frac{2^{n...

Đã gửi bởi Stranger411 on 16-08-2012 - 10:06 trong Số học

Bài trên lâu lắm rồi :D Mình mở rộng bằng căn nguyên thủy tí nữa cho nó mạnh :D

Mở rộng: Tìm tất cả các số nguyên $n>1$ sao cho tồn tại duy nhất số nguyên $a$ với $0 < a < n!$ sao cho:
\[n!|{a^n} + 1\]

BÀI TOÁN: Xác định tất cả các số nguyên $n> 1$ sao cho $\frac{2^{n}+1}{n^{2}}$ là một số nguyên.

Bài này còn 2 cách giải nữa bằng căn nguyên thủy và LTE ;)



#347129 Tìm số nguyên $n> 1$ sao cho $\frac{2^{n...

Đã gửi bởi Stranger411 on 16-08-2012 - 09:52 trong Số học

BÀI TOÁN: Xác định tất cả các số nguyên $n> 1$ sao cho $\frac{2^{n}+1}{n^{2}}$ là một số nguyên.

Bài này ko cần phải dùng đến cấp của 1 số đâu :)

Bổ đề 1: Cho các số nguyên $m,n$ và $a>1$. Ta có: $\gcd \left( {{a^m} - 1,{a^n} - 1} \right) = {a^{\gcd \left( {m,n} \right)}} - 1$
Bổ đề 2: Nếu ${3^b}|{2^a} - 1 \Rightarrow {3^{b - 1}}|a$

Lời giải bài toán:
+ Khi $n=1$, bài toán thỏa mãn.
+ Khi $n>1 \Rightarrow n$ lẻ.
Gọi $p$ là ước nguyên tố lẻ nhỏ nhất của $n$ nên $\gcd \left( {p - 1,n} \right) = 1$.
Ta có: $p|{2^n} + 1|{2^{2n}} - 1$
Theo định lí Ferma nhỏ, ta có: $p|{2^{p - 1}} - 1$.
Áp dụng bổ đề 1, ta được: $p|\gcd \left( {{2^{p - 1}} - {{1,2}^{2n}} - 1} \right) = {2^{\gcd \left( {2n,p - 1} \right)}} - 1$
mà $\gcd \left( {2n,p - 1} \right) \leqslant 2 \Rightarrow p|3 \Rightarrow p = 3$
Đặt $n = {3^k}d$. Dùng bổ đề 2, ta có: ${3^{2k}}|{n^2}|{2^{2n}} - 1 \Rightarrow {3^{2k - 1}}|n \Rightarrow k = 1$
(*) Nếu $d>1$. Gọi $q$ là ước nguyên tố nhỏ nhất của $d$ nên $q \ge 5$.
Lập luận tương tự như trên, ta có: $q=7$
Vậy nên $7|n|{2^n} + 1$. Điều này vô lí vì ${2^n} + 1 \equiv 2,3,5(\bmod 7)$.
(*) Nếu $d=1$, ta có: $n=3$.
Vậy $n=1$ và $n=3$ thỏa mãn đề bài.



#346650 ${p_{k + 1}}|{p_1} + {p_2} +...

Đã gửi bởi Stranger411 on 14-08-2012 - 11:48 trong Số học

Cho số nguyên $n \ge 2$.
Chứng minh rằng tồn tại 1 bộ hoán vị ${p_1},{p_2}, \ldots ,{p_n}$ của $1,2, \ldots ,n$ sao cho ${p_{k + 1}}|{p_1} + {p_2} + \ldots + {p_k}$ với $k = 1,2, \ldots ,n-1$



#346612 $\sum\limits_{k = 1}^{p - 1} {\l...

Đã gửi bởi Stranger411 on 13-08-2012 - 23:32 trong Số học

Cho p là số nguyên tố .CMR $\sum\limits_{k = 1}^{p - 1} {\left\lfloor {\frac{{{k^3}}}{p}} \right\rfloor } = \frac{{(p + 1)(p - 1)(p - 2)}}{4}$

Khuya rồi !! Lên Vmf chém thử vài bài cho vui :D

Với $1 \leqslant k \leqslant p - 1$thì $\left\{ \begin{gathered}
{k^3} \equiv 0(\bmod p) \\
{\left( {p - k} \right)^3} \equiv - {\left( k \right)^3} \\
\end{gathered} \right.$
Nên $\left( {\frac{{{k^3}}}{p} - \left\lfloor {\frac{{{k^3}}}{p}} \right\rfloor } \right) + \left( {\frac{{{{\left( {p - k} \right)}^3}}}{p} - \left\lfloor {\frac{{{{\left( {p - k} \right)}^3}}}{p}} \right\rfloor } \right) = 1$.
Từ đó, ta được:
$\begin{gathered}
\sum\limits_{k = 1}^{p - 1} {\left\lfloor {\frac{{{k^3}}}{p}} \right\rfloor } = \sum\limits_{k = 1}^{p - 1} {\frac{{{k^3}}}{p}} - \sum\limits_{k = 1}^{p - 1} {\left( {\frac{{{k^3}}}{p} - \left\lfloor {\frac{{{k^3}}}{p}} \right\rfloor } \right)} \\
= \frac{1}{p}\sum\limits_{k = 1}^{p - 1} {{k^3}} - \frac{1}{2}\sum\limits_{k = 1}^{p - 1} {\left( {\frac{{{k^3}}}{p} - \left\lfloor {\frac{{{k^3}}}{p}} \right\rfloor } \right)} + \left( {\frac{{{{\left( {p - k} \right)}^3}}}{p} - \left\lfloor {\frac{{{{\left( {p - k} \right)}^3}}}{p}} \right\rfloor } \right) \\
= \frac{1}{p}{\left( {\frac{{p\left( {p - 1} \right)}}{2}} \right)^2} - \frac{{p - 1}}{2} = \frac{{\left( {p + 1} \right)\left( {p - 1} \right)\left( {p - 2} \right)}}{4} \\
\end{gathered}$

ps Chú Nguyenta98 post trước mình 4 phút.



#346577 Tìm các số nguyên dương $a,b,c$ sao cho $\frac{a^{2}+b^{2...

Đã gửi bởi Stranger411 on 13-08-2012 - 21:29 trong Số học

$\Rightarrow 2a^2+2ab+2b^2 \vdots p \Rightarrow 4a^2+4ab+4b^2 \vdots p \Rightarrow (2a+b)^2+3b^2 \vdots p$

$c \equiv - a - b(\bmod p) \Rightarrow p|a^2 + ab + b^2 \Rightarrow p|{(2a + b)^2} + 3{b^2}$
$ \Rightarrow \left( {\frac{{ - 3}}{p}} \right) = 1$. Và điều này vô lí vì $p \equiv 2(\bmod 3)$.
Vậy không tồn tại $a,b,c$ thỏa mãn bài toán. $\blacksquare$

Bạn xem lại @@!
Bài mình và bài Tạ để đi đến $p|{(2a + b)^2} + 3{b^2}$ để suy ra vô lí mà (:|

Bạn nói lại xem mình sai chỗ nào (:|